Anda di halaman 1dari 37

KEY POINTS:

The most common syndrome resulting from viral CNS infection is meningitis, which can be defined as inflammation of the subarachnoid space and meninges without direct involvement of brain parenchyma. Neurotropic viruses usually spread to the CNS by the hematogenous route but may also spread along nerves via retrograde axonal transmission.

VIRAL MENINGITIS AND ENCEPHALITIS


Roberta L. DeBiasi, Kenneth L. Tyler ABSTRACT
Hundreds of human viral pathogens exhibit a tropism for the central nervous system (CNS). In the case of some viruses, involvement of the CNS is the predominant feature of the resulting illness whereas in others involvement of the CNS is a rare complication of a more generalized illness (Corboy and Tyler, 2000). Infection with these viruses may result in several recognizable neurological syndromes, depending upon the specific elements of the CNS that are preferentially attacked. The most common syndrome resulting from viral CNS infection is meningitis, which can be defined as inflammation of the subarachnoid space and meninges without direct involvement of brain parenchyma (Hammer and Connolly, 1992). In contrast, the syndrome of encephalitis is characterized by viral infection of brain tissue itself. Although the same viruses are responsible for inducing both meningitis and encephalitis, individual viruses may more commonly produce one or the other syndrome (Johnson, 1998; Johnson, 1996). Viruses often simultaneously affect both meninges and brain parenchyma as so-called meningoencephalitis. According to data from the US Centers for Disease Control and Prevention (CDC), over 100,000 cases of aseptic meningitis occur annually in the United States, the majority of which are of viral etiology. Approximately 20,000 cases of encephalitis occur in the United States each year, most of which are mild. In this chapter, viral agents that cause meningitis and encephalitis will be addressed, as well as the differential diagnosis of these viral diseases in normal and abnormal hosts. This chapter concentrates, for the most part, on the most common causes of these diseases in North America.

58

VIRUS ENTRY AND SPREAD TO THE CENTRAL NERVOUS SYSTEM Viruses initially gain entrance into the host by penetration of mucosal, skin, gastrointestinal, or urogenital barriers. Once within the host, access to the CNS is via one of two routes: hematogenous or neuronal spread. Hematogenous dissemination, the more common of the two, generally results from primary viral replication near the site of entry, followed by secondary viremia and seeding of distant sites, such as endothelial cells of meningeal capillaries with secondary passage to the subarachnoid space, or direct seeding of choroid plexus. Viruses can

subsequently spread from the choroid plexus to cerebrospinal fluid (CSF), to ependymal cells lining the ventricles, and into brain tissue, depending on the specific tropism of the particular virus and the host immune response. Many enteroviruses (EVs) cause CNS disease by this route after primary replication within the gastrointestinal tract. Alternatively, viruses spread to the CNS via peripheral nerve endings by means of retrograde axonal transmission; such transmission is characteristic of rabies, for example, but may also occur with herpes simplex virus (HSV), varicella-zoster virus (VZV), and poliovirus (Figure 3-1). Neurotropic

Copyright @ American Academy of Neurology. Unauthorized reproduction of this article is prohibited.

KEY POINT:

Acute disseminated encephalomyelitis is a condition in which widespread demyelination occurs in a monophasic pattern, often following a definite or suspected viral infection or immunization. It is presumably the result of an immunemediated reaction against a component of normal brain such as myelin.

FIGURE 3-1

Pathogenesis of viral infection of the central nervous system. HIV = human immunodeficiency virus; HSV = herpes simplex virus.
Modified from Hammer SM, Connolly KJ. Viral aseptic meningitis in the United States: clinical features, viral etiologies, and differential diagnosis. Cur Clin Top Infect Dis 1992;12:125.

59

viruses usually spread to the CNS by the hematogenous route but may also spread along nerves via retrograde axonal transmission (Tyler, 2001). ENCEPHALITIS, ACUTE DISSEMINATED ENCEPHALOMYELITIS, AND ENCEPHALOPATHY When evaluating patients with altered mental status, the clinician must dis-

tinguish infectious encephalitis from encephalopathy as well as postinfectious or parainfectious immune-mediated neurological syndromes such as acute disseminated encephalomyelitis (ADEM) (Kennedy, 2004; Whitley and Gnann, 2002). Although viruses more commonly cause disease at the time of host entry and invasion, postinfectious or parainfectious neurological conditions,

Copyright @ American Academy of Neurology. Unauthorized reproduction of this article is prohibited.

" VIRAL MENINGITIS AND ENCEPHALITIS

60

presumably immune mediated, appear after many viral infections. These will be discussed in detail later. In brief, ADEM is a condition in which widespread demyelination occurs in a monophasic pattern, often following a definite or suspected viral infection or immunization. It is presumably the result of an immune-mediated reaction against a component of normal brain, such as myelin (Davis, 2000). Multiple sclerosis (MS) is also in the differential diagnosis of ADEM, although MS affects an older age group, and the pattern of demyelination in MS is not typically monophasic. The following features may help distinguish ADEM from acute encephalitis. ADEM tends to occur preferentially in children and often occurs within a month of a vaccination or an uncomplicated prodromal illness such as a childhood exanthem, upper respiratory infection, or gastroenteritis. Importantly, neurological illnesses typically begin at the end of a prodromal illness rather than preceding or occurring concomitantly with it. Symptoms are monophasic and develop over a few days. Multiple focal signs develop, with optic nerve, spinal cord, and cerebellar involvement noted more frequently than in viral encephalitis. Rapid progression to coma is more common in ADEM than in most viral encephalitides. Magnetic resonance imaging (MRI) findings distinguish ADEM from encephalitis and include the presence of disseminated white matter lesions, with a high T2 and low T1 signal, which enhance following gadolinium. The presence of coincident enhancement of the lesions suggests that they are of recent and similar age, as enhancement resulting from inflammatory blood-brain barrier disruption usually resolves within 6 weeks of an acute lesion. The presence of myelin basic protein and oligoclonal bands in

the CSF all favor the diagnosis of ADEM, although these may also be present in patients with some types of encephalitis. Conversely, patients with ADEM do not have detectable viral infection in the CNS, and as a result, viral cultures of the CSF or brain tissue and CSFpolymerase chain reaction (PCR) studies are usually negative. An additional condition that must be distinguished from infectious encephalitis is encephalopathy, whether of metabolic, toxin-mediated, or other origin. Although mental status is also altered in encephalopathy, as it is in infectious encephalitis, patients generally do not exhibit fever or headache, and the CSF is usually normal. Seizures and focal neurological signs are uncommon, and patients exhibit steady deterioration of mental status rather than the more commonly seen fluctuating mental status of encephalitis. DIAGNOSIS Although this review addresses the primary agents of viral meningitis and encephalitis, many infectious (eg, bacterial, viral, fungal, parasitic, rickettsial, myoplasmal), and noninfectious etiologies must be considered in any patient who presents with findings suggestive of CNS disease. Several groups have recently attempted to identify the relative frequencies with which these occur, using a battery of serological and molecular diagnostic techniques. In the California Encephalitis Project, one of the largest studies to date, a viral etiology was confirmed or probable in 9% of 334 cases, with fewer cases due to bacterial (3%) and parasitic (1%) etiologies. Noninfectious etiologies were identified in 10% and nonencephalitic infections in 3%. A possible etiology was found in 12%. Despite extensive testing, the etiology remained unexplained in 62% of cases

Copyright @ American Academy of Neurology. Unauthorized reproduction of this article is prohibited.

61

FIGURE 3-2

Algorithm for management of patient with suspected viral central nervous system infection. Continued on next page

Copyright @ American Academy of Neurology. Unauthorized reproduction of this article is prohibited.

" VIRAL MENINGITIS AND ENCEPHALITIS

KEY POINTS:

In acute disseminated encephalomyelitis, optic nerve, spinal cord, and cerebellar involvement are noted more frequently than in viral encephalitis. The presence of myelin basic protein and oligoclonal bands in the cerebrospinal fluid all favor the diagnosis of acute disseminated encephalomyelitis, although these may also be present in patients with some types of encephalitis.

FIGURE 3-2

Continued. ADEM = acute disseminated encephalomyelitis; CT = computed tomography; MRI = magnetic resonance imaging; CSF = cerebrospinal fluid; PCR = polymerase chain reaction; HSV = herpes simplex virus; IgM = immunoglobulin M; WNV = West Nile virus; DFA = direct fluorescent antibody (test); VZV = varicella-zoster virus; VDRL = Venereal Disease Research Laboratory (test); AFB = acid-fast bacilli; EBV = Epstein-Barr virus; IVDA = intravenous drug abuse; CTFV = Colorado tick fever virus; LCMV = lymphocytic choriomeningitis virus; HIV = human immunodeficiency virus.

62

(Glaser et al, 2003). A larger study that utilized an extensive panel of CSF PCR assays for detection of 11 viruses detected an etiological agent in 14% of 3485 specimens tested (Huang et al, 2004). Although these disparate etiologies produce symptoms that may appear similar, each disease has unique features that suggest it as causative in a given patient. The most common viruses identified in North America causing meningitis and encephalitis are noted in Table 3-1. They are listed in the relative order of frequency with which they occur, and their relative propensity to cause meningitis, encephalitis, and/or postinfectious encephalomyelitis is indicated. Additional causes of viral encephalitis that should be considered in patients who have returned from

abroad are noted separately in Table 3-2. The nonviral etiologies of CNS disease are summarized in Table 3-3, which includes distinguishing features of each agent or disease (Rubeiz and Roos, 1992; Tyler, 2001). In any given patient, the most urgent distinctions to be drawn center on identification of bacterial meningitis and/or HSV encephalitis. Effective antimicrobial therapy is available for both with the capability of drastically reducing associated morbidity and mortality when administered in a timely fashion. It is imperative to institute immediately appropriate empirical antibacterial and antiviral therapies while initiating a detailed diagnostic evaluation. Once these diagnoses have been excluded by Grams stain and culture data of CSF and HSV-PCR studies (see

Copyright @ American Academy of Neurology. Unauthorized reproduction of this article is prohibited.

TABLE 3-1

Primary Causes of Viral Meningitis and Encephalitis in North America Postinfectious Acute Disseminated Encephalomyelitis

Agent
Nonpolio enteroviruses Arboviruses (United States and Canada) Togaviruses Flavivirus Echovirus Coxsackievirus

Meningitis
*** ***

Encephalitis
* *

St Louis encephalitis virus (SLE) West Nile virus Powassan

* *

** ** **

Alphavirus

Eastern equine (EEE) Western equine (WEE) Venezuelan equine (VEE) Colorado tick fever California (La Crosse) Jamestown Canyon Snowshoe hare

* * ** ** * * * * ** * * * *

*** ** * * ** * * ** * ** ** * ** *** * *

Reoviridae: orbivirus Bunyavirus

Herpes viruses

HSV-1 HSV-2 VZV CMV EBV HHV-6 Herpes B virus

63

Lymphocytic choriomeningitis virus (LCMV) Mumps virus Human immunodeficiency virus (HIV) Rabies virus Measles virus

**

** **

* *

*** *

* ***

Continued on next page

Copyright @ American Academy of Neurology. Unauthorized reproduction of this article is prohibited.

" VIRAL MENINGITIS AND ENCEPHALITIS

TABLE 3-1

Continued
Postinfectious Acute Disseminated Encephalomyelitis
* * ** *

Agent
Rubella virus Poliovirus (now eradicated from Western hemisphere) Adenovirus Vaccinia Influenza Parainfluenza Rotavirus Parvovirus B-19

Meningitis

Encephalitis

** *

? * * *

? *

* *

HSV-1 = herpes simplex virus l; HSV-2 = herpes simplex virus 2; VZV = varicella-zoster virus; CMV = Cytomegalovirus; EBV = Epstein-Barr virus; HHV-6 = human herpesvirus 6. * Occasional ** Common *** Very common ? Unknown

64

below), a staged approach to identify other nonbacterial causes of meningitis or encephalitis can be initiated. A tiered algorithmic approach to this process is summarized in Figure 3-2, suggesting immediate evaluation for treatable lifethreatening etiologies, followed by second-tier evaluation for other common agents for which identification is less urgent. Finally, third-tier evaluation for more rare etiologies is indicated for patients in whom no diagnosis has been made. A history of unusual exposures, travel, or specific symptoms may clearly alter the diagnostic approach to certain patients in whom atypical etiologies are more likely. GENERAL FEATURES OF VIRAL MENINGITIS AND ENCEPHALITIS The hallmark of both viral meningitis and encephalitis is the acute onset of a febrile illness accompanied by

headache and, often, nuchal rigidity. With many forms of encephalitis, altered mental status, disorientation, behavioral and speech disturbances, and focal or diffuse neurological signs such as hemiparesis or seizures may occur, and these symptoms help distinguish it from meningitis, in which they are generally absent (Table 3-4) (Tyler, 2001). Historical points that may help pinpoint a specific viral infection include the season of year (eg, fall for EVs, summer/fall for arboviruses); travel history (eg, regional arboviruses, viruses with foreign distributions); knowledge of diseases prevalent within the community (eg, enteroviral or arboviral outbreaks); history of animal exposure (eg, rabies, lymphocytic choriomeningitis virus [LCMV]) or mosquito or tick (arboviruses) exposure. It is helpful to review preceding

Copyright @ American Academy of Neurology. Unauthorized reproduction of this article is prohibited.

TABLE 3-2 Agent


Nipah virus Filovirus

Additional Causes of Viral Encephalitis Resulting From Foreign Exposures Geographical Distribution
Indonesia Ebola Marburg Africa

Arbovirus Togavirus Mosquito-borne Eastern equine Venezuelan equine St Louis Japanese B Murray Valley West Nile Ilheus Rocio Tick-borne complex Far Eastern Central European Kyansur Forest Louping Ill Negishi Russian spring-summer Bunyavirus Tahyna Caribbean, South America (plus United States) Central and northern South America (plus United States) Caribbean, Central and northern South America (plus United States) Japan, China, Southeast Asia, India Australia, New Guinea Africa, Mideast, parts of Europe (plus United States) South and Central America Brazil Eastern Russia Eastern and Central Europe, Scandinavia India England, Scotland, Northern Ireland Japan Eastern Europe, Asia Czech Republic, Slovakia, Yugoslavia, Italy, Southern France Finland East Africa Many underdeveloped countries

65

Inkoo Rift Valley Rabies

or concurrent illnesses or symptoms that have occurred in the days to weeks leading up to the presenting disorder (eg, ADEM, VZV, Epstein-Barr virus [EBV], HSV, mumps). Sexual activity and intravenous (IV) drug use comprise other potentially important historical points (ie, patients with the

possibility of human immunodeficiency virus [HIV]). All patients should, of course, undergo a complete general medical and neurological examination with special attention to alteration of mental state, papilledema, cranial nerve deficits, abnormal reflexes, or focal weakness. Although many viruses

Copyright @ American Academy of Neurology. Unauthorized reproduction of this article is prohibited.

" VIRAL MENINGITIS AND ENCEPHALITIS

TABLE 3-3 Etiology


Reoviridae: orbivirus Infectious

Diseases That Can Masquerade as Viral Meningitis or Encephalitis Suggestive Features


Bacterial Parameningeal focus (sinusitis, intracranial abscess) Partially treated bacterial meningitis Lyme disease Very mild pleocytosis, focal neurological abnormalities Prior antibiotic treatment, right shifted cerebrospinal fluid Tick exposure, arthritis, appropriate geography, erythema migrans Very high protein, hypoglycorrhachia Conjunctival suffusion, jaundice Chronic course Farm animal exposure Gastrointestinal complaints Cat exposure, adenopathy Brain stem encephalitis Exposure history, bradycardia Usually immunocompromised patient Southwestern United States exposure, pulmonary symptoms Pulmonary nodules Midwest, pulmonary symptoms Immunocompromised patient Immunocompromised patient Retinitis, cat exposure Calcified lesions Fresh water: Naegleria Exposure history Leukopenia, thrombocytopenia, hyponatremia, petechial rash See above Exposure to sheep, pulmonary disease Precedent pulmonary symptoms

Tuberculosis Leptospirosis Syphilis Brucella Whipples disease Bartonella (catscratch) Listeria Typhoid fever Fungal Cryptococcus Coccidioides

Histoplasma Blastomycosis Candida Nocardia

66

Parasitic

Toxoplasma Cysticercosis Amoebic Malaria (Plasmodium falciparum)

Rickettsial

Rocky Mountain spotted fever Ehrlichia Coxiella burnetii (Q fever)

Mycoplasma

Continued on next page

Copyright @ American Academy of Neurology. Unauthorized reproduction of this article is prohibited.

TABLE 3-3 Etiology

Continued
Suggestive Features
Acute disseminated encephalomyelitis (ADEM) Connective-tissue disorders Systemic lupus erythematosus (SLE) Sarcoidosis Uveomeningitic syndromes Intracranial tumors and cysts Drugs Nonsteroidal anti-inflammatory drugs (NSAIDs), antibiotics, immunomodulators, anticonvulsants Behc ets Characteristic magnetic resonance imaging findings Malar rash, multisystem organ involvement Hilar adenopathy, erythema nodosum Genital/oral ulcers, uveitis Recurrent episodes, dermal sinus tract Exposure history

Parainfectious

Noninfectious

Intracranial hemorrhage Encephalopathy Toxic or metabolic

Characteristic neuroimaging findings

cause diffuse cerebral involvement, the tropism of viruses for different cell types within the CNS may lead to characteristic neurological findings. For example, the predisposition of HSV for the temporal lobe may lead to clinical findings of aphasia, anosmia, and temporal lobe seizures. Although focal signs should always suggest HSV, other viruses may present as focal encephalitis and must also be considered (Table 3-5). Since many of the features of aseptic meningitis and encephalitis are common to the majority of viral etiologies, the presence of unusual physical findings can better help define a specific viral diagnosis (Gutierrez and Prober, 1998). The presence of mucous membrane or skin abnormalities may be especially helpful in this respect since different viruses tend to

cause characteristic enanthems or exanthems (Table 3-6). Other unusual findings such as alopecia, arthritis, lymphadenopathy, myocarditis, orchitis, pneumonia, urinary difficulty, or retinitis are also suggestive in specific instances (Table 3-7) (Roos, 1998; Tyler, 1984). Evaluation of the CSF and performance of appropriate imaging studies are essential features of the laboratory evaluation. Blood studies may be helpful in certain conditions. For example, leukopenia and thrombocytopenia are often seen with rickettsial infections as well as certain arboviral infections such as Colorado tick fever virus. Serum serologies are useful in many conditions. The CSF should be obtained and analyzed as quickly as possible. It is helpful to send routine studies on a

67

Copyright @ American Academy of Neurology. Unauthorized reproduction of this article is prohibited.

" VIRAL MENINGITIS AND ENCEPHALITIS

portion of the CSF, reserving one tube for specific studies such as virusspecific serologies and PCR, as suggested by the pattern of results from routine studies (discussed individually below) (DeBiasi and Tyler, 1999). In most forms of viral CNS disease, the CSF contains a mild to moderate pleocytosis, from a few to up to 1000

white blood cells (WBCs)/mm3; a slightly lower range is usual in viral encephalitis (up to hundreds of cells). A lymphocytic or mononuclear WBC predominance is common, distinguishing this syndrome from bacterial etiologies in which predominance of polymorphonuclear cells is characteristic. If the CSF is examined early in the clinical

TABLE 3-4 Category

Characteristic Findings in Viral Meningitis and Encephalitis Meningitis


Mental status Seizures Neurological examination Fever Usually normal Uncommon Usually normal; cranial nerve deficits possible Usually less than 408C

Encephalitis
Altered: confusion, delirium, lethargy, stupor, coma Often present: focal or generalized Focal findings common Often high

Physical examination

Laboratory Cerebrospinal fluid White blood cell (WBC) count WBC shift 10 mm3 to 1000/mm3, usually less than 300/mm3 Early polymorphonuclear (PMN) predominance, followed by mononuclear shift in 24 to 48 hours (in West Nile virus, PMNs may persist) Usually normal (except mumps, or lymphocytic choriomeningitis viruslow) Normal or mildly elevated 50 mg/100mL to 100 mg/100mL Usually normalmay see meningeal enhancement Normal Up to several hundred WBCs/mm3. Occasionally acellular Same

68

Glucose

Same

Protein

Same

Imaging

Magnetic resonance imaging/computed tomography Electroencephalogram

Usually abnormal

Usually abnormal with diffuse bilateral background slowing and/or epileptiform activity Perivascular inflammation, neuronal and glial necrosis, edema, inclusion bodies

Brain pathology

Normal

Copyright @ American Academy of Neurology. Unauthorized reproduction of this article is prohibited.

disease, however, polymorphonuclear cells may also be present in nonbacterial (including viral) infections of the CNS. In viral CNS infection, however, this pattern shifts within 8 to 24 hours toward the more usual mononuclear predominance. Infection with EV and HSV may be more likely to produce this pattern than other viruses. A polymorphonuclear predominance that persists

is typical of HIV-associated Cytomegalovirus polyradiculomyelitis, as well as West Nile virus meningoencephalitis; with these exceptions, a persistently polymorphonuclear pleocytosis is inconsistent with viral etiologies and requires careful exclusion of bacterial and nonviral processes. The CSF glucose is usually normal in viral meningitis and encephalitis, although

TABLE 3-5 Agent

Differential Diagnosis of Viral Encephalitis With Focal Cerebral Involvement Characteristic Pattern
Magnetic resonance imaging with increased signal intensity in orbitofrontal and temporal lobe areas on T2-weighted images Electroencephalogram with pseudoperiodic, focal or unilateral, high-amplitude complexes in one or both temporal lobes (periodic lateralizing epileptiform discharges)

Herpes simplex virus

Human herpesvirus 6

May mimic herpes simplex virus (temporal lobe involvement) Focal seizures, hemiparesis, or focal cranial nerve deficits)

Varicella-zoster virus

May follow zoster eruption (days to months) or occur without any skin eruption May occur with disseminated zoster May occur in setting of primary varicella-zoster virus infection (1 week after onset of rash)

Enterovirus Arboviruses: LaCrosse St Louis Powassan Measles viruspostmeasles encephalomyelitis

Primarily in infants and hypogammaglobulinemic patients

Focal neurological signs in 20% of cases May mimic herpes simplex virus

69

Multifocal neurological signs during convalescence from measles (within 2 weeks of eruption of rash) Magnetic resonance imaging with multiple hyperintense areas on T2-weighted images in subcortical white matter

Subacute measles encephalitis Lymphocytic choriomeningitis virus JC virus

Occurs in immunocompromised patients with latent period of 1 to 20 months between the measles illness and encephalitis Mild encephalitis with only rare fatalities Progressive multifocal leukoencephalopathy in patients with depressed cell-mediated immunityusually insidious onset

From Roos KL. Pearls and pitfalls in the diagnosis and management of central nervous system infectious diseases. Semin Neurol 1998;18:185196. Modified with permission from Thieme Medical Publishers.

Copyright @ American Academy of Neurology. Unauthorized reproduction of this article is prohibited.

" VIRAL MENINGITIS AND ENCEPHALITIS

TABLE 3-6

Skin/Mucous Membrane Findings Suggesting Specific Viral Central Nervous System Diseases

Exanthem or Mucous Membrane Change


Vesicular eruption

Viral Agent
Enterovirus Herpes simplex Varicella-zoster virus (VZV)

Specific Changes
Hand-foot-and-mouth diseasemacules/papules/ vesicles on palms, soles, buttocks Grouped small (3 mm) vesicles on an erythematous base Zoster: vesicles in dermatomal distribution Primary VZV: multiple vesicles, papules, pustules in various stages of eruption Diffuse maculopapular eruption following ampicillin treatment Diffuse maculopapular erythematous eruption beginning on face/chest and extending downward Roseola: Diffuse maculopapular eruption following 4 days of high fever Maculopapular rash in 50% Occasionally occurs with lymphadenopathy

Maculopapular eruption

Epstein-Barr virus Measles Human herpesvirus 6 Colorado tick fever Lymphocytic choriomeningitis virus West Nile virus

Diffuse truncal eruptionmay spread to limbs and face Many types of rash Confluent erythema over cheeks (slapped cheek) followed by lacy, reticular rash over extremities (late) Rare stocking-glove syndromepurpuric lesions on distal extremities Herpanginavesicles on soft palate Pharyngoconjunctival fever Conjunctivitis Conjunctivitis with pharyngitis (see above)

Erythema multiforme Confluent macular rash Purpura Pharyngitis

Herpes simplex Mycoplasma* Parvovirus Parvovirus Enterovirus Adenovirus

70

Conjunctivitis

St Louis encephalitis Adenovirus

* Mycoplasma is not a virus.

hypoglycorrhachia is well recognized in LCMV and mumps meningitis. Even in these conditions, the glucose is usually greater than 25 mg/dL. Values below 25 mg/dL should suggest the possibility of bacterial or fungal infection and sarcoid or carcinomatous meningitis. CSF protein is generally normal

or slightly elevated in viral meningitis; viruses that tend to present with elevated CSF protein include HSV (late) and EV (early). Table 3-4 summarizes these common CSF findings (Davis, 2000). Table 3-8 outlines the utility of CSF and serum PCR, serology, and culture

Copyright @ American Academy of Neurology. Unauthorized reproduction of this article is prohibited.

KEY POINTS:

TABLE 3-7 Finding


Alopecia Arthritis

Other Specific Findings Associated With Viruses Causing Central Nervous System Disease Viruses
LCMV LCMV, Parvovirus LCMV, Colorado tick fever LCMV, mumps Mumps Cytomegalovirus, Epstein-Barr virus West Nile virus, St Louis encephalitis virus, varicella-zoster virus, herpes B virus, LCMV, human T-lymphotropic virus, rabies Enterovirus (mumps, LCMV) Mumps (LCMV, Epstein-Barr virus) Colorado tick fever, LCMV, rabies Mumps (LCMV) Influenza, parainfluenza Cytomegalovirus Arboviruses (West Nile virus and others) St Louis encephalitis virus, varicella-zoster virus, herpes B virus, LCMV West Nile virus, rabies

Biphasic illness Lymphadenopathy Mastitis Mononucleosis Myelitis

Myocarditis/pericarditis Orchitis/oophoritis Paraesthesias Parotitis Pneumonia Retinitis Tremors Urinary problems Weakness

When evaluating patients with altered mental status, the clinician must distinguish infectious encephalitis from encephalopathy as well as postinfectious or parainfectious immunemediated neurological syndromes such as acute disseminated encephalomyelitis. In any given patient, the most urgent distinctions to be drawn center on identification of bacterial meningitis and/ or herpes simplex virus encephalitis. The hallmark of both viral meningitis and encephalitis is the acute onset of a febrile illness accompanied by headache and, often, nuchal rigidity.

LCMV = Lymphocytic choriomeningitis virus.

A
for the most common forms of viral meningitis and encephalitis. In some viral infections such as nonpolio EV, LCMV, mumps, and rabies it is possible to culture virus directly from CSF. However, this is time-consuming, and sensitivity of culture methods varies greatly with virus strain and laboratory technique. Some viruses, on the other hand (eg, EBV and HSV), can only rarely be cultivated from CSF, even under the best conditions. Culture of other specimens such as throat/respiratory washings, rectal swabs, skin vesicles, urine, saliva, or blood may be helpful in some cases but may not always prove CNS involvement, as unrelated shedding from prior infection is always possible. Acute and convalescent serological diagnosis is also potentially useful in some viral infections, but the utility of this method is limited by the length of time required to firmly establish acute infection (weeks in many cases), as well as the limited specificity of many serologies. One exception is in the case of acute EBV infection, in which the presence of immunoglobulin M (IgM) to viral capsid antigen is extremely suggestive of acute infection. Serology remains the most generally available means of diagnosing arboviral infection. The presence of specific intrathecal

71

Copyright @ American Academy of Neurology. Unauthorized reproduction of this article is prohibited.

" VIRAL MENINGITIS AND ENCEPHALITIS

TABLE 3-8

Laboratory Evaluation for Identification of Specific Viral Agents Causing Acute Meningitis and Encephalitis Polymerase Chain Reaction Serum

Virus

Serology*

Culture Specimens

CSF Serum CSF Throat Rectal Blood CSF Other


++ ++ + +
y

Enterovirus Poliovirus Arboviruses Herpes simplex virus 1 and herpes simplex virus 2 Varicella-zoster virus Cytomegalovirus Epstein-Barr virus Human herpesvirus 6 Lymphocytic choriomeningitis virus Adenovirus

+ + +
z

++ +

+ +

++ ++

+/ ++

++ +/ Skin vesicle, brain tissuex Skin vesiclex Urine Urine

++ +/

++

++ + + +/

+ ++ + +/ +

+ +

++ ++ ++ ++

+ +/ +/

+ +/ + +
z

+ Rare +/ +z ++

++ +/ ++

Urine

+/

+/

++

Conjunctival swab, stool, electron microscopy Urine, saliva Urine

Mumps Measles Human immunodeficiency virus Rabies

++

+ +
z

+ + +

+ +

++ +

+/ + +/

++

++

++

Saliva, brain tissue, nuchal skin biopsy, corneal impressionx

Influenza

++

+/
k

+/ +/ + ++

++

++x ++

72

Parainfluenza Parvovirus West Nile

++ Extremely effective for diagnosis. + Effective test for diagnosis. +/ Variable effectiveness. Not useful. CSF = cerebrospinal fluid. *Serological indexing, which compares CSF to serum-specific antibody levels in reference to total CSF, and serum albumin or total immunoglobulin may be required for definitive diagnosis. Fourfold rise in immunoglobulin G (IgG) from acute to convalescent specimens or single positive immunoglobulin M (IgM) may also be diagnostic. y Serology may be difficult to interpret in vaccinated patients or postviral setting. In these patients, presence of IgM indicates active or reactivated viral disease rather than past immunity. z Not widely available but can be sent to research laboratories. x Direct fluorescent antibody test (DFA) on tissues or secretions is more rapid and may be more sensitive than culture methods. DFA has replaced Tzanck prep, which is less sensitive and specific. k Most helpful in immunocompromised patients in whom viremia is prolonged/persistent.

Copyright @ American Academy of Neurology. Unauthorized reproduction of this article is prohibited.

synthesis of antibody, directed against a particular viral pathogen, is highly suggestive of its role in CNS disease; these tests are, however, time-consuming and not highly sensitive. The most promising development for diagnosis of CNS infection is the PCR technique, which has the capability of detecting minute amounts of viral DNA or RNA in CSF or other body fluids (DeBiasi and Tyler, 2004; DeBiasi and Tyler, 2000). PCR has improved the rapidity and accuracy of diagnosis of viral CNS infections, enhanced the understanding of pathogenesis, and helped identify additional, previously unknown infectious causes of CNS disease. By making quick and precise diagnoses, appropriate treatments can be instituted and unnecessary or invasive investigations can be avoided. PCR for HSV and EV has had the most dramatic impact on management of patients with viral meningitis and encephalitis to date (see below). MRI and computed tomography (CT) scan can also provide useful information in the evaluation of CNS infection. Although CT scans may demonstrate abnormalities in the setting of encephalitis, with hypodensity of involved brain parenchyma and irregular zones of contrast enhancement, such changes are often slow to develop and are frequently nonspecific. MRI with gadolinium contrast is by far more sensitive and is the neuroimaging procedure of choice in most cases. Changes in acute encephalitis may include edema and abnormalities of the basal ganglia, cortex, and gray-white matter junction. Focal abnormalities on MRI may suggest particular diagnoses, such as T1hypointense and T2-hyperintense signal in orbitofrontal and temporal lobe areas in HSV encephalitis. MRI may also be helpful in distinguishing encephalitis from ADEM, in which prominent areas of demyelination (often symmetrical) of spinal cord, white matter, and basal

ganglia are common, as has already been discussed. Positron emission tomography and single-photon emission CT imaging are newer modalities that may provide even more sensitive functional and metabolic data in the setting of viral encephalitis. However, these studies are costly, complex, and not readily available in many areas (Steiner et al, 2005). The three most common causes of viral meningitis and encephalitis in North America are EVs, arboviruses (particularly West Nile virus), and HSV. In adolescents and adults herpes simplex virus 1 (HSV-1) more commonly causes encephalitis, whereas HSV-2 commonly causes meningitis. Less common, but not rare, etiologies of viral meningitis and/or encephalitis include LCMV, HIV, other herpes viruses (eg, human herpesvirus 6 [HHV-6], EBV, Cytomegalovirus, VZV), and rabies. Enteroviruses Since the eradication of poliovirus from the Western Hemisphere, nonpolio EVs are the most commonly implicated group of viruses in patients with viral meningitis (Rotbart, 1997). The EV family comprises nearly 70 different serotypes within the Picornavirus family. They can be subgrouped into the polioviruses, coxsackie viruses A and B, echoviruses, and the newer sequentially numbered EVs. The strains most commonly isolated in aseptic meningitis are coxsackie A9, B3, B4, B5, and echovirus 4, 6, 7, 9, 11, 18, and 30. Typically, EV infections are either asymptomatic or result in mild disease, and fewer than one in 500 infections results in aseptic meningitis. More than 75,000 cases of EV meningitis occur in the United States each year. Spread of infection is by the fecal-oral and, occasionally, respiratory routes. Outbreaks tend to cluster in the late summer and early fall and may be associated with pharyngitis and

KEY POINTS:

Historical points that may help pinpoint a specific viral infection include the season of year (fall for enteroviruses, summer/fall for arboviruses), travel history (regional arboviruses, viruses with foreign distributions), knowledge of diseases prevalent within the community (enteroviral or arboviral outbreaks), history of animal exposure (rabies, lymphocytic choriomeningitis virus), or mosquito/tick (arboviruses) exposure. The tropism of viruses for different cell types within the CNS may lead to characteristic neurological findings. The presence of unusual physical findings can better help define a specific viral diagnosis.

73

Copyright @ American Academy of Neurology. Unauthorized reproduction of this article is prohibited.

" VIRAL MENINGITIS AND ENCEPHALITIS

KEY POINTS:

In most forms of viral CNS disease, the cerebrospinal fluid contains a mild to moderate pleocytosis, from a few to up to 1000 WBCs/mm3; a slightly lower range is usual in viral encephalitis (up to hundreds of cells). A polymorphonuclear predominance that persists is typical of HIV-associated Cytomegalovirus polyradiculomyelitis, as well as West Nile virus meningoencephalitis; with these exceptions, a persistently polymorphonuclear pleocytosis is inconsistent with viral etiologies and requires careful exclusion of bacterial and nonviral processes.

Case 3-1
A 4-year-old girl presents during the autumn with headache, nuchal rigidity, and vomiting. She is noted to have skin lesions on her palms and soles consisting of tender papules and clear vesicles, with surrounding erythema, and is found to have scattered mildly erythematous oral ulcerations on her soft palate, gingiva, tip of her tongue, and inner lip. CSF contains 200 WBCs/mm3, with 50% polymorphonuclear cells and 50% mononuclear cells, a protein content of 60 mg/dL, and glucose of 60 mg/dL. The CSF-PCR study is positive for EV. Comment. EVs are responsible for approximately 80% of aseptic meningitis cases, especially those occurring in the autumn. The pediatric population is overrepresented. Patients may have other physical findings suggesting enteroviral disease, such as the exanthem/enanthem of hand-foot-mouth syndrome. CSF may demonstrate a pleocytosis with polymorphonuclear predominance in the first 48 hours of illness, which subsequently shifts to mononuclear cells. CSF reverse transcriptase (RT)-PCR is more sensitive than culture for detection of EVs. Serious morbidity from this illness is rare, except in neonates and hypogammaglobulinemic patients who may develop overwhelming sepsislike illness or evolve into a chronic meningoencephalitis. Recently, enterovirus 71 outbreaks associated with more severe illness, including brain stem encephalitis and paralytic poliomyelitislike presentations.

74

gastrointestinal symptoms such as anorexia, vomiting, or diarrhea. Other findings suggestive of enteroviral infection include the enanthem of herpangina or the exanthem of handfoot-mouth syndrome. Onset of symptoms is usually sudden with fever usually under 40oC as a consistent finding. Although EV is the most common cause of aseptic meningitis in adults as well as in the pediatric population, children are epidemiologically overrepresented as victims of enteroviral infection (Case 3-1). Fortunately, enteroviral meningitis occurring beyond the neonatal period in normal hosts is only rarely associated with severe disease or subsequent neurological deficits (Sawyer, 1999). Although more commonly the etiological agent in aseptic meningitis, EVs may also cause encephalitis, particularly in immunodeficient patients with agammaglobulinemia and in neonates. EV strain 70 has been implicated most frequently in instances of encephalitis.

In addition to being one of the major causative agents of hand-foot-andmouth disease, EV strain 71 has also been reported in association with more severe neurological illness, including poliomyelitislike paralysis and fulminant brain stem encephalitis. Epidemics of severe EV71 disease have recently occurred in children in Taiwan (50 deaths in 1998) and Malaysia (30 deaths in 1997). In neonates, EV meningoencephalitis is generally part of an overwhelming sepsislike illness, with up to 10% mortality. Infection of hypogammaglobulinemic patients commonly leads to a chronic and progressive meningoencephalitis; these patients are treated with IV immunoglobulin therapy and, more recently, antivirals active against EVs (see below). Until recently, recovery of EVs from the CSF was the primary means of establishing the diagnosis of CNS enteroviral disease. Recovery of nonpolio EV from throat or rectal swabs is also suggestive but not diagnostic in a

Copyright @ American Academy of Neurology. Unauthorized reproduction of this article is prohibited.

patient with aseptic meningitis, since shedding from a previous, unrelated EV infection may occur in the upper respiratory tract for 1 to 3 weeks and in the feces for up to 8 weeks following infection. The clinical utility of obtaining a viral culture is further limited by the amount of time required for these agents to grow (days to weeks), relatively low sensitivity (65% to 75%), as well as the poor cultivability of some EV serotypes. The development of a reliable PCR for the detection of EV has been extremely useful in this regard (Ramers et al, 2000). The PCR primers and probes are directed against the 50 noncoding region of the viral genome, which is highly conserved among almost all enteroviral strains. These primers and probes are designed for reverse transcription of the viral RNA genome combined with PCR (RT-PCR). The newest generation of the enteroviral PCR on CSF has 95% to 96% sensitivity and 100% specificity for the 67 known strains of EV. For reasons that are not clear, CSF PCR is often negative in the setting of EV71 disease, but enteroviral genome is readily detectable from throat, rectal, and serum specimens. Conventional RTPCR methods can produce results within 24 hours, and a more recently developed colorimetric assay can be performed in approximately 5 hours. The availability of EV RT-PCR should allow for the more rapid diagnosis of this common illness and limit unnecessary hospitalizations and empiric antibacterial therapy. Although treatment has been supportive to date, resolution of the atomic structure of EVs by x-ray crystallographic studies has led to the design of new antiviral therapy. These studies demonstrate a deep cleft or canyon in the center of each protomeric unit of the viral capsid into which the susceptible host cells specific cellular receptor for the EV fits.

The drug pleconaril was specifically designed to fit into this cleft, and it therefore can block the cell/receptor interaction required for viral entry into the host. A double-blind, placebocontrolled multicenter trial study of 130 patients aged 14 to 65 years with enteroviral meningitis demonstrated resolution of headache and other meningitis symptoms 2 days earlier ( P=.04) in pleconaril-treated (200 mg 3 times a day for 7 days) as compared with placebo-treated patients. Preliminary outcomes of potentially lifethreatening enteroviral infections treated with compassionate-use pleconaril (including chronic enteroviral meningoencephalitis in agammaglobulinemic patients, enteroviral encephalitis, and neonatal EV sepsis) have been reported and demonstrate beneficial effect on clinical, virologic, laboratory, and radiologic parameters. Unfortunately, pleconaril is not currently in active production nor easily obtained. Although pleconaril was not submitted to the US Food and Drug Administration for approval for treatment of enteroviral meningitis or other lifethreatening enteroviral infections, this drug should be considered in the case of more severe or overwhelming disease, as in neonates and agammaglobulinemic patients, in the case that a compassionate-release program is reinstated in the future. Herpes Simplex Virus Types 1 and 2 Herpes viruses are double-stranded DNA viruses that share the ability to remain latent within the human host. They are commonly implicated in infections of the CNS. In adults, HSV-2 is classically associated with aseptic meningitis, including recurrent episodes, and it also accounts for about 10% of cases of HSV-related encephalitis. Conversely, HSV-1 is more commonly

KEY POINTS:

In some viral infections such as nonpolio enterovirus, lymphocytic choriomeningitis virus, mumps, and rabies it is possible to culture virus directly from cerebrospinal fluid. Acute and convalescent serological diagnosis is potentially useful in some viral infections, but the utility of this method is limited by the length of time required to firmly establish acute infection (weeks in many cases), as well as the limited specificity of many serologies. The most promising development for diagnosis of CNS infection is the polymerase chain reaction technique, which has the capability of detecting minute amounts of viral DNA or RNA in cerebrospinal fluid or other body fluids.

75

Copyright @ American Academy of Neurology. Unauthorized reproduction of this article is prohibited.

" VIRAL MENINGITIS AND ENCEPHALITIS

KEY POINTS:

MRI with gadolinium contrast is by far more sensitive than CT and is the neuroimaging procedure of choice in most cases in the evaluation of CNS infection. Focal abnormalities on MRI may suggest particular diagnoses such as T1hypointense and T2hyperintense signal in orbitofrontal and temporal lobe areas in herpes simplex virus encephalitis. The three most common causes of viral meningitis and encephalitis in North America are enteroviruses, arboviruses (particularly West Nile virus), and herpes simplex virus.

associated with encephalitis in adults and less frequently with aseptic meningitis. This distinction is, however, not invariable, since neonates are more likely to develop meningoencephalitis with either HSV-1 or HSV-2. Aseptic meningitis. At the time of their first episode of genital herpes (generally HSV-2), approximately 36% of women and 11% of men have symptoms of meningitis, including fever, headache, and nuchal rigidity. Of these patients, 20% will go on to have recurrent episodes of meningitis. The meningitis may result from viremic spread to the CNS or, alternatively, by direct invasion of the CNS via ascent from sacral nerve roots. When aseptic meningitis occurs in this setting, genital lesions are present an average of 1 week before CNS symptoms. Other neurological symptoms such as paraesthesias, urinary retention, and transverse myelitis have been

described. Recovery is usually complete, although persistent or recurrent radiculopathy may occur. Treatment with acyclovir in the setting of primary infection is reasonable as it may shorten the overall course of this form of HSV-2 infection, but it has no impact on latency or genital recurrences. CSF viral cultures are invariably negative during recurrent episodes of meningitis in these patients, although the virus may be isolated during the first (primary) episode of meningitis. As in Case 3-2, diagnosis depends on PCR of the CSF, which currently has a sensitivity of 95% and specificity of 100% in experienced laboratories, with a turnaround time generally within 48 hours. A large subset of patients with the syndrome of benign recurrent aseptic meningitis has been found to have HSV-specific DNA detectable in their CSF (DeBiasi and Tyler, 2000). In one

Case 3-2
A 30-year-old man presents with fever, headache, and nuchal rigidity but otherwise appears well. There is no history of travel, rash, or precedent illness, although he has had a new sexual partner for the last 2 months, and he has noticed itching and painful urination starting 1 week ago. The neurological examination is normal. The CSF evaluation demonstrates 150 predominantly mononuclear cells, with a protein of 45 mg/dL and glucose of 60 mg/dL. Penile examination shows multiple crusted erosions on an erythematous base with a few intact vesicular lesions also present. Culture of the lesions, as well as CSF-PCR, is positive for HSV. Comment. Aseptic meningitis occurs in 11% to 36% of patients with primary genital herpes (usually HSV-2), more commonly in women than men. Most patients with their primary episode of genital herpes are clinically asymptomatic; fewer than 10% of those with HIV-associated aseptic meningitis report a history of genital lesions. Systemic symptoms (eg, fever, malaise, and headache) are more common in women than in men. Localized itching, pain, and dysuria can occur. Genital lesions include multiple vesicles with red bases that form pustules, rupture, and crust over. Treatment with acyclovir may shorten the duration of symptoms but has no impact on latency or frequency of subsequent episodes. Neurological sequelae are unusual, but up to 20% of patients experience recurrent episodes of meningitis, which likely reflects a subset of patients with benign recurrent lymphocytic meningitis (Mollarets meningitis).

76

Copyright @ American Academy of Neurology. Unauthorized reproduction of this article is prohibited.

KEY POINTS:

Case 3-3
A 60-year-old man is brought to the hospital with aphasia, right-sided weakness, fever, and confusion. A lumbar puncture demonstrates CSF with 400 red blood cells (RBCs)/mm3, 30 WBCs/mm3 (predominantly mononuclear cells), glucose of 70 mg/dL, and protein of 60 mg/dL. The electroencephalogram (EEG) shows periodic high-voltage spike-wave activity from the left temporal region, and an MRI documents T1-hypointense and T2-hyperintense signal in cortex and gray-white matter junction in the left temporal lobe. PCR of CSF for HSV is positive. Comment. The main clue to a diagnosis of HSV in adults and adolescents is evidence of focal disease, especially referable to the temporal lobe. The presence of RBCs in the CSF and mildly elevated protein are characteristic of HSV encephalitis but are not specific. Rapid institution of IV acyclovir therapy is essential; treatment reduces mortality from 70% in untreated patients to 19%. Many patients have significant sequelae, despite appropriate therapy.

Enteroviruses are responsible for approximately 80% of aseptic meningitis cases, especially those occuring in the autumn. Outbreaks of enterovirus meningitis tend to cluster in the late summer and early fall and may be associated with pharyngitis and gastrointestinal symptoms such as anorexia, vomiting, or diarrhea. Enteroviral meningitis occuring beyond the neonatal period in normal hosts is only rarely associated with severe disease or subsequent neurological deficits. Enteroviruses may also cause encephalitis, particularly in immunodeficient patients with agammaglobulinemia and in neonates.

of the largest series to date, 85% of patients had HSV detectable by PCR, of which 91% were HSV-2. These patients are predominantly female and experienced three to nine attacks of meningitis over a period of 2 to 23 years. Of these patients, only 23% have a history of genital herpes, none with lesions concurrent with their most recent episodes of meningitis. Encephalitis. As exemplified by Case 3-3, HSV-1 (and rarely HSV-2) is typically associated with focal encephalitis in adolescents and adults, and it carries a 70% mortality if untreated; over 97% of survivors are left with permanent neurological deficits (Baringer, 2000). It is the most common cause of sporadic, acute, focal encephalitis in the United States and accounts for approximately 10% of all cases of encephalitis in the United States annually. Early recognition is essential because of the availability of effective antiviral therapy. HSV-1 encephalitis occurs throughout the year without seasonal preference and in all age groups, although one third of the cases develop in patients younger than 20 years and 50% in those older than 50 years. It is possible that this bimodal age distribution reflects pri-

mary infection in younger patients and reactivation of latent virus in older individuals. The majority of cases of HSV encephalitis occurs in previously healthy individuals. About 30% to 60% of patients report a brief prodromal illness prior to the onset of encephalitis, the most common being upper respiratory and gastrointestinal illnesses. Fever, headache, and altered mental status are invariably present and the patient may exhibit confused or bizarre behavior. Vivid auditory, olfactory, or visual hallucinations are experienced by 20% of patients. Seizures, focal, generalized, or both, occur in 75% of patients. A variety of focal signs and symptoms appear in 75% of patients that may help localize the disease to the inferior frontal and medial temporal lobes. The most common finding is hemiparesis or motor weakness, which occurs in 45% of patients. Aphasia of any degree of severity is particularly common (75%). Impaired memory, especially for events of the recent past, may suggest medial temporal lobe involvement. Other findings occasionally encountered include cranial nerve abnormalities, movement disorders, eye movement abnormalities,

77

Copyright @ American Academy of Neurology. Unauthorized reproduction of this article is prohibited.

" VIRAL MENINGITIS AND ENCEPHALITIS

KEY POINTS:

Nearly 70 serotypes of enteroviruses (predominantly coxsackievirus and echovirus) cause viral meningitis and, less commonly, encephalitis. Aseptic meningitis occurs in 11% to 36% of patients with primary genital herpes (usually HSV-2), more commonly in women than men. Diagnosis of herpes simplex virus aseptic meningitis depends on polymerase chain reaction of the CSF, which currently has a sensitivity of 95% and specificity of 100% in experienced laboratories, with a turnaround time generally within 48 hours. A large subset of patients with the syndrome of benign recurrent aseptic meningitis has been found to have herpes simplex virus specific DNA detectable in their CSF.

78
A

frontal release signs, and autonomic instability. CSF abnormalities are generally similar to those found in other forms of viral encephalitis. Protein elevations occur in 90% but rarely exceed 200 mg/dL. Hypoglycorrhachia is decidedly unusual in HSV disease (less than 5%). RBCs are found in the CSF in 50% to 75% of cases but rarely exceed 500 cells/mm3. The presence of CSF protein elevation or of RBCs is suggestive of the diagnosis but is not unique to HSV encephalitis. Rarely, patients with HSV encephalitis fail to have a CSF pleocytosis. A normal CSF cell count occurs in less than 5% of patients but may occur with greater frequency in immunocompromised patients, including those infected with HIV. EEG abnormalities may also provide clues to diagnosis since 90% of patients have focal or generalized slowing. The presence of periodic sharp-wave complexes in temporal leads superimposed on a slow-amplitude background is highly suggestive of HSV. The sharp waves may be monophasic or polyphasic; they last from 100/ms to 1500/ms and occur at 1-second to 5-second intervals. This pattern is only rarely seen after the first 2 weeks of illness. MRI (Figure 3-2) may be normal early in the course of HSV encephalitis, but within days focal edema and hemorrhage are usually evident. MRI is much more sensitive than CT scan, but eventually 60% to 100% of patients will show some abnormality on CT scan as well. In the past, cerebral biopsy with virus isolation had been the gold standard for diagnosis of HSV encephalitis. However, this has been replaced by detection of HSV DNA by PCR in the CSF (Lakeman and Whitley, 1995). If done with optimal techniques in an experienced laboratory, the specificity is 100% and the sensitivity 95% in most recent studies. However, multi-

ple groups have reported the occurrence of false-negative (initially negative, subsequently positive) CSF PCR for HSV, leading to the realization that testing performed within the first 72 hours of symptoms may not be sensitive. A negative PCR result in a patient with clinically compatible disease and no alternative identified etiology should not deter treatment for this treatable cause of encephalitis. The CSF PCR remains positive for approximately 2 weeks following onset of illness, but sensitivity declines rapidly after that point. Brief (less than 72 hours) antiviral therapy does not significantly reduce the likelihood of a positive PCR, but the amount of viral DNA declines with prolonged therapy. Almost all patients have negative PCR tests after the completion of a 14day course of acyclovir. CSF serology to detect the presence of intrathecal production of antibody may provide supportive evidence of HSV infection. Serological studies are less sensitive and less specific than CSF PCR but may be useful in late diagnosis when viral nucleic acid is no longer detectable. More recently developed type-specific HSV serology (immunoglobulin G [IgG] and IgM) may have improved sensitivity and specificity and may be a useful adjunct to diagnosis. Treatment with acyclovir (at a dose of 30 mg/kg/d IV divided every 8 hours for 14 to 21 days in adults, and 60 mg/kg/d divided every 8 hours for 21 days in neonates, is the management of choice (see below). Neonatal herpes simplex virus disease. Infants who acquire HSV, (HSV-2 more commonly than HSV-1) during the newborn period develop infection of the CNS in over 50% of cases. Infection most frequently occurs at the time of birth from the maternal genital tract (85%) but may also occur as a result of transplacental transmission (5%) or in the postpartum period (10%).

Copyright @ American Academy of Neurology. Unauthorized reproduction of this article is prohibited.

KEY POINTS:

Case 3-4
A 3-week-old infant, born at full term by uncomplicated vaginal delivery, presents with poor feeding, lethargy, and a temperature of 38.38C. Findings from physical examination are unremarkable and without focal neurological deficits. Liver enzymes are mildly elevated; leukopenia and thrombocytopenia are evident in the blood count. The CSF contains 200 WBCs (100% mononuclear) with a protein of 120 mg/ dL and glucose of 60 mg/dL. Blood, urine, and CSF cultures are obtained; and ampicillin, cefotaxime, and acyclovir are administered IV. CSF PCR for HSV is positive. Comment. HSV disease of the newborn presents as one of three distinct entities: (1) Disseminated disease (20%), which presents as a nonspecific multiorgan sepsislike syndrome with meningoencephalitis, pneumonitis, hepatitis and coagulopathy; (2) CNS disease (35%), characterized by meningoencephalitis (with detectable HSV by CSF PCR), with or without accompanying classic vesicular skin lesions; and (3) skin, eye, mouth disease (45%), in which HSV is limited to mucosal and/or skin surfaces only. Although infants with CNS disease may present with relatively mild symptoms of low-grade fever and lethargy, mortality approaches 50% in untreated infants. With appropriately instituted acyclovir therapy, mortality has been reduced to 14%, but severe neurological sequelae still occur in two thirds of treated, surviving infants.

Herpes simplex virus 1 (and rarely herpes simplex virus 2) is typically associated with focal encephalitis in adolescents and adults and carries a 70% mortality if untreated; over 97% of survivors are left with permanent neurological deficits. The most common finding in herpes simplex virus encephalitis is hemiparesis or motor weakness, which occurs in 45% of patients. Red blood cells in herpes simplex virus are found in the CSF in 50% to 75% of cases but rarely exceed 500 cells/mm3.

CNS disease occurs in primarily two forms: disseminated disease, with multiorgan involvement such as lung and liver in addition to the CNS, or disease localized to the CNS with or without accompanying characteristic skin vesicles (Case 3-4). The route to the brain in infants with multiorgan dissemination is likely hematogenous and associated with a diffuse encephalitic process. Alternatively, nondisseminated neuronal transmission may occur with disease limited to the CNS only (unitemporal or bitemporal). Importantly, infants (usually aged 2 to 10 weeks) with HSV meningoencephalitis may present with isolated skin, eye, and mouth vesicles in absence of fever, and little or no symptomatology suggesting CNS disease. In this age group, it is imperative to evaluate the CSF for evidence of pleocytosis and HSV DNA despite an otherwise normal-appearing infant. Treatment and outcome of neonatal HSV disease are described below (Kimberlin, 2004; Whitley and Kimberlin, 1999).

Arboviruses The rubric arbovirus is an umbrella term defining viral agents that are transmitted to humans by mosquito and tick vectors. They persist in nature in complex cycles involving birds and mammals, which serve as reservoirs of disease. When transmitted to humans, they can cause fever, headache, meningitis, encephalitis, and even death. The three primary families into which arboviruses are divided are: (1) togaviruses (subdivided in turn into flaviviruses and alphaviruses), (2) reoviruses, and (3) bunyaviruses. All are single-stranded RNA viruses (Table 3-1). The most common arboviruses affecting humans in North America are West Nile virus, St Louis encephalitis (SLE) virus (flaviviruses), California (La Crosse) encephalitis virus (bunyavirus), and eastern equine encephalomyelitis (alphavirus). Western equine encephalitis, Venezuelan equine encephalitis, Colorado tick fever virus, and Powassan virus are rare but important arboviruses

79

Copyright @ American Academy of Neurology. Unauthorized reproduction of this article is prohibited.

" VIRAL MENINGITIS AND ENCEPHALITIS

KEY POINTS:

A normal CSF cell count occurs in less than 5% of patients with herpes simplex virus encephalitis but may occur with greater frequency in immunocompromised patients, including those infected with human immunodeficiency virus. The presence on EEG of periodic sharp-wave complexes in temporal leads superimposed on a slow amplitude background is highly suggestive of herpes simplex virus. Cerebral biopsy with virus isolation had been the gold standard for diagnosis of herpes simplex virus encephalitis. However, this has been replaced by detection of herpes simplex virus DNA by polymerase chain reaction in the CSF.

80

causing disease in North America. These virusesexistinwell-definedgeographical regions, which vary from strain to strain. The age of targeted victims and virulence also vary among arboviruses. Most cases occur in the summer months since transmission depends on the bite of mosquito or tick. Differences among these are summarized in Table 3-9 (Calisher, 1994; Lowry, 1997). West Nile virus. Although never observed in the Western Hemisphere prior to 1999, West Nile virus has emerged as the most common cause of epidemic meningoencephalitis in North America. First isolated in 1937 from a febrile patient in Uganda, West Nile virus is widely distributed throughout the world. The virus has recently exhibited a propensity for increased neurovirulence and has caused major outbreaks of meningoencephalitis in Eastern Europe and the Middle East throughout the last decade. The first cases of West Nile virus disease in North America were reported in New York state in the summer of 1999, with the simultaneous occurrence of an unusual number of deaths of exotic birds at the Bronx Zoo as well as crows in the New York City metropolitan area, soon followed by an unusually high incidence of encephalitis in humans (66 cases and 9 deaths). Analysis of sequences of genome fragments isolated from dead birds and mosquitoes (by RT-PCR) revealed West Nile virus, which was 99.8% identical to sequences analyzed from human isolates/autopsy specimens, as well as a 1998 isolate from the brain of a dead Israeli goose. As the virus has spread prolifically within bird reservoirs (greater than 200 species) and an unprecedented number of mosquito species in subsequent years, the number and geographical range of human cases in the United States have greatly expanded: 4156 cases/284 deaths in 2002; 9864 cases (2866 neuroinvasive)/264 deaths

in 2003; and 2539 cases (1142 neuroinvasive)/100 deaths in 2004. Currently, West Nile virus has been identified in all parts of the United States with the exception of Washington, Alaska, and Hawaii. Transmission to humans occurs predominantly following mosquito bite, but cases have also resulted from bloodproduct transfusion, organ transplantation, percutaneous (occupational), as well as intrauterine and breastfeeding exposures. Eighty percent of individuals infected with West Nile virus remain completely asymptomatic. Twenty percent develop a self-limited flulike illness following a 2- to 14-day incubation period, consisting of fever, myalgia, headache, and gastrointestinal disturbance, with nonspecific maculopapular rash in 50% of cases. Neuroinvasive disease occurs in less than 1% of cases (1 in 150 infected individuals). Neuroinvasive presentations are varied and include aseptic meningitis, meningoencephalitis, and acute flaccid paralysis syndrome (poliomyelitislike). Brain stem encephalitis, movement disorders, cranial neuropathies, polyneuropathy/radiculopathy, and optic neuritis are also recognized West Nile virus neurological presentations. The proportion of neuroinvasive disease manifesting as meningitis as opposed to encephalitis or myelitis has varied greatly within a given epidemic season and locale. Persons more than 50 years of age are at highest risk (20-fold increase) for developing meningoencephalitis (Case 3-5). Despite being equally susceptible to West Nile virus infection, neuroinvasive disease is very uncommon in pediatric patients, although all three forms of neuroinvasive disease have been reported in this age group. Several clinical features are distinctive of West Nile virus neurological infection. Muscle weakness has been a prominent finding in patients with

Copyright @ American Academy of Neurology. Unauthorized reproduction of this article is prohibited.

either meningitis or meningoencephalitis (30% to 50%). Other distinctive findings have included hyporeflexia (30%) and cranial neuropathies (20%), most commonly facial palsy. Postural or kinetic tremor has been reported in up to 80% of cases of meningitis and meningoencephalitis. Myoclonus has also been appreciated in 20% and 40%, respectively. Parkinsonian features, including rigidity, bradykinesia, and postural instability, have been noted in up to 75% of patients with meningoencephalitis. Patients with West Nile virus acute flaccid paralysis syndrome have acute onset and rapid progression of asymmetrical flaccid weakness with associated hyporeflexia or areflexia in involved limbs. Acute flaccid paralysis may occur in isolation or in combination with meningitis or meningoencephalitis. Weakness may occur in a single limb or any combination of the four extremities, and respiratory insufficiency may also occur. Sensation is preserved. Bowel and bladder dysfunction occurs in one third of patients. Unlike meningoencephalitis, the West Nile virus acute flaccid paralysis syndrome does not have a predilection for the elderly and has been reported in all age groups. Electromyography studies demonstrate reduced amplitudes of compound muscle action potentials with normal amplitudes of sensory nerve action potentials. Follow-up studies 3 or more weeks later may show denervation changes. Pathological studies have demonstrated acute anterior poliomyelitis in some cases. Typical CSF findings in West Nile virus neuroinvasive disease include pleocytosis (polymorphonuclear or lymphocytic predominance) with elevated protein and normal glucose levels. Features of the pleocytosis that appear to be unique to West Nile virus include a prolonged (up to 1 week) polymorphonuclear predominance and a plas-

macytoid appearance to the lymphocytes. MRI is unremarkable in the majority of cases; however, abnormalities of the brain stem and deep nuclei (thalamus, basal ganglia, substantia nigra) as well as leptomeningeal enhancement have been reported in approximately 30% of cases of meningoencephalitis. Patients with acute flaccid paralysis syndrome may show signal abnormalities within the cord, but most do not. EEG abnormalities occur in the majority of patients with encephalitis; 60% to 100% demonstrate diffuse irregular slowing, whereas focal sharp waves or seizure activity are uncommon. Serological and molecular methods for diagnosis are discussed below (in the context of other arboviral infections). Treatment of West Nile virus (and all arboviral) disease is generally supportive, and person-to-person transmission does not occur. Several treatment modalities for West Nile virus CNS disease are currently being evaluated, including passive immunization using IV immunoglobulin with high titer to West Nile virus (multicenter randomized trial), interferon alpha, and antisense oligomer constructs. Ribavirin appears to have limited clinical efficacy despite demonstrated efficacy against West Nile virus in vitro. These treatments may prove to be most useful for affected immunocompromised patients, in whom disease is most severe. An effective formalin-inactivated equine vaccine exists but cannot be used in humans. Much progress toward the development of a human West Nile virus vaccine has been made, and clinical trials of a chimeric vaccine, in which West Nile virus genes are inserted into the genetic background of another flavivirus, are underway. The overall mortality rate for West Nile virus infection is 2% to 7%; the mortality attributable to encephalitis is estimated at 12% to 15% and is higher in the elderly

KEY POINTS:

Multiple groups have reported the occurrence of falsenegative (initially negative, subsequently positive) CSF polymerase chain reaction for herpes simplex virus, leading to the realization that testing performed within the first 72 hours of symptoms may not be sensitive. Treatment with acyclovir (at a dose of 30mg/kg/d intravenously divided every 8 hours for 14 to 21 days in adults, and 60 mg/kg/d divided every 8 hours for 21 days in neonates, is the management of choice for herpes simplex virus encephalitis. Infants who acquire herpes simplex virus, (HSV-2 more commonly than HSV-1) during the newborn period develop infection of the CNS in over 50% of cases.

81

Copyright @ American Academy of Neurology. Unauthorized reproduction of this article is prohibited.

" VIRAL MENINGITIS AND ENCEPHALITIS

TABLE 3-9 Agent


West Nile

Details of North American Arboviruses Geographical Distribution


Throughout United States

Reservoir
Birds

Vector
Mosquito

St Louis

Throughout United States but greatest prevalence in Texas, Florida, and Ohio-Mississippi River Valley Midwest and Northeast United States, southern Canada Atlantic and Gulf coasts, Great Lakes region Western United States and Canada Texas and Florida North central United States, Eastern Canada United States and Canadian Rocky Mountains

Birds

Mosquito

California (La Crosse) Eastern equine Western equine Venezuelan equine Powassan Colorado tick fever

Chipmunk, squirrel, small mammals Birds Birds and small mammals Horses, small animals Squirrel, porcupine, groundhog Chipmunk, squirrel, rodent

Mosquito Mosquito Mosquito Mosquito Tick Tick

82

population (35%). Long-term outcome and sequelae from West Nile virus meningoencephalitis are not fully known at this time, but several reports have documented prolonged fatigue, myalgia, residual tremor, and parkinsonism. Patients with acute flaccid paralysis have the worst overall prognosis and often have permanent disability (Petersen and Marfin, 2002; Tyler, 2004). St Louis encephalitis virus. SLE virus remains one of the most important (although only third most common) causes of epidemic encephalitis in North America since it carries a high rate of neurological sequelae (25%) and mortality (up to 20%). Since 1964, 4478 reported human cases of SLE have been reported, with an average of 128 cases reported annually. Cases of SLE are widespread throughout the United States, occur-

ring periodically in focal outbreaks (up to 3000 cases per year in large outbreaks, such as 1975). The most recent outbreak of SLE occurred in New Orleans, Louisiana, in 1999 with 20 reported cases. The risk of encephalitis following infection with SLE virus increases with age, with almost 90% of elderly patients developing encephalitis (Case 3-6). Humans infected with SLE may exhibit a wide array of clinical manifestations, but most are asymptomatic. If symptomatic, fully 40% may have only fever and headache as their complaint. Urinary complaints occur in up to 25% of patients with SLE and cranial nerve palsies in 20%. California (La Crosse) virus encephalitis. California (La Crosse) virus encephalitis is a much milder disease, with only rare incidence of neurological sequelae and less than

Copyright @ American Academy of Neurology. Unauthorized reproduction of this article is prohibited.

TABLE 3-9 Season

Continued
Group Affected
Severe disease in adults over 50 years

Mortality
12% to 15% of patients with severe disease (but less than 1% have severe disease) 2% to 20%

Neurological Sequelae
Common (varies with presentation)

Peak in June to September (Cases reported year-round) June to August

Adults over 50 years

25% to 50% mild Less than 10% severe

June to September June to August June to September Rainy season May to December Spring/summer March to September

Children Children Infants, adults over 50 years Adults

Less than 1% 50% to 80% Adults: 3% to 5% Infants: 10% to 20% Less than 1% Rare

Rare 80% (especially young) Adults: 5% Infants: 50% Rare 50% Rare

Children and adults

Less than 1%

1% mortality. An average of 70 cases per year have been reported in the United States since 1964. Ninety percent of cases occur in children less than 15 years of age. In areas endemic for California (La Crosse) virus encephalitis (ie, the Midwest and northeast United States, southern Canada), a high proportion of the population appears to have been infected. Seropositivity rates as high as 12% to 18% have been reported in some studies, and many experts believe that these numbers underestimate the true prevalence in endemic populations. Most infections are asymptomatic. When illness does occur, onset is abrupt. Patients present with fever, chills, and headache and may also have photophobia, abdominal pain, or upper respiratory illness symptoms. Forty percent have seizures.

Eastern equine encephalitis. Eastern equine encephalitis virus produces the most severe illness among arboviruses, with 50% to 70% mortality and neurological sequelae among 80% of survivors. Fortunately, this virus infection occurs only infrequently in the United States. Only four cases from four states were reported to the CDC in 1998 (median of four cases per year from 1964 to 1998). The disease occurs almost exclusively along the Atlantic and Gulf coasts, as well as in the Great Lakes region. Onset is typically abrupt. Patients present with high fever, convulsions, and rapid mental deterioration, with progression to coma. Young children are particularly susceptible and have a high rate of neurological sequelae if they survive. Western equine encephalitis. No cases of Western equine encephalitis

83

Copyright @ American Academy of Neurology. Unauthorized reproduction of this article is prohibited.

" VIRAL MENINGITIS AND ENCEPHALITIS

KEY POINTS:

The rubric arbovirus is an umbrella term defining viral agents that are transmitted to humans by mosquito and tick vectors. Most cases of arbovirus occur in the summer months since transmission depends on the bite of mosquito or tick. St Louis encephalitis virus remains one of the most important (although only third most common) causes of epidemic encephalitis in North America since it carries a high rate of neurological sequelae (25%) and mortality (up to 20%). Cases of St Louis encephalitis are widespread throughout the United States, occurring periodically in focal outbreaks.

Case 3-5
A 70-year-old man presents in midsummer with headache, confusion, left leg weakness, and diplopia, following a 3-day history of high fever (398C), nausea, and nonspecific rash. His history is otherwise unremarkable, other than multiple mosquito bites during the previous 3 weeks. Findings from the physical examination are remarkable for altered mental status, hyporeflexia of the left lower extremity, and a kinetic tremor. An MRI scan of the brain is normal. Analysis of CSF reveals 100 WBCs/mm3 with polymorphonuclear predominance, glucose 60 units, and protein 90 mg/dL. CSF for West Nile virus IgM is positive. Comment. West Nile virus has emerged as the most common cause of epidemic meningoencephalitis in North America and the most common of the arboviral encephalitides in the United States. Elderly and immunocompromised persons are at disproportionate risk for severe neuroinvasive disease. Neuroinvasive presentations include aseptic meningitis, meningoencephalitis, and poliomyelitislike paralysis. Motor weakness is a prominent associated finding in 50% of cases of West Nile virus meningoencephalitis. Cranial neuropathies, movement disorders, gastrointestinal complaints, and rash are also seen in a significant proportion of cases (see discussion above). Treatment is supportive, although IV immune globulin with high titer to West Nile virus and other modalities are being investigated as potential therapeutic interventions.

have been reported in the United States since 1994, with the last epidemic occurring in 1987 in Colorado. The single largest outbreak occurred in 1941, with over 3000 human cases recognized. Equine surveillance continues in many western states and Canada, since equine cases precede human

cases by 2 to 3 weeks. Infants appear to be particularly susceptible to CNS disease; 20% of patients are less than 1 year of age. Venezuelan equine encephalitis. The strain of Venezuelan equine encephalitis virus endemic in the southern United States only rarely causes

84

Case 3-6
A 70-year-old man presents in July with low-grade fever, headache, and urinary incontinence followed by generalized seizures. He had spent the prior week fishing with his grandchildren at a wooded lake. The CSF viral culture is negative, but CSF PCR is positive for SLE virus, and acute/ convalescent titers show a fourfold rise in IgG for SLE virus. In the next several weeks, a notable increased incidence of aseptic meningitis and febrile seizures occurs in local emergency departments. Comment. SLE virus is transmitted by mosquito vector, most commonly during the summer months. More severe disease is noted in elderly patients, with up to 90% developing frank encephalitis. Although less common than California (La Crosse) virus, the geographical distribution is less restricted, and widespread outbreaks have occurred throughout most of the United States. Neurological sequelae result in 25% of infected patients.

Copyright @ American Academy of Neurology. Unauthorized reproduction of this article is prohibited.

encephalitis. The majority of cases are presumed to be asymptomatic; a flulike syndrome of fever, malaise, headache, myalgia/arthralgia, and gastrointestinal symptoms may be noted. Encephalitis is found in about 6% of these patients and has less than 1% mortality with rare neurological sequelae. Colorado tick fever. Colorado tick fever virus is likely under-recognized as a cause of aseptic meningitis and encephalitis during the summer months among residents of the United States and Canadian Rocky Mountain regions (average 200 reported cases per year). After an incubation period of 3 to 6 days, symptoms appear abruptly and include 5 to 8 days of high fever, chills, joint/ muscle pain, severe headache, ocular pain, conjunctival injection, and nausea. A transitory petechial or maculopapular rash is present occasionally. Colorado tick fever virus classically causes a biphasic illness with remission of initial symptoms for 2 to 3 days, followed by a relapse of 2 to 3 days in 50% of patients. Leukopenia and thrombocytopenia are common. The vast majority of cases are mild and uncomplicated with an excellent prognosis. Antibody to Colorado tick fever virus is not detected until 1 to 2 weeks after the illness; viremia is present for months. Virus may be isolated from blood and CSF. A PCR has been developed for diagnostic use in endemic regions. Powassan virus. Powassan virus is an extremely rare and severe cause of encephalitis, distributed in the north central United States and eastern Canada. In reported cases, the onset is sudden with headache, high fever, and seizures common. Prodromal symptoms include sore throat, sleepiness, disorientation, and headache 1 week following a tick bite. Most patients have focal lesions, and sequelae such as hemiplegia, recurrent severe headache, and damage to upper cervical cord have been reported in nearly 50% of survivors.

Additionally, many other arboviruses exist that are endemic in foreign countries and must be considered in the returning traveler with infectious CNS disease (Table 3-2). Diagnosis of arboviral disease is dependent upon specific serological studies in the setting of mosquito or tick exposure in an appropriate geographical region. The presence of arbovirus-specific IgM in CSF is diagnostic of neuroinvasive disease (since IgM antibodies do not cross the blood-brain barrier), indicating intrathecal synthesis of antibody from local antigen exposure. IgM antibody capture enzymelinked immunoassay (ELISA) from acute CSF is the most sensitive and specific means of making the diagnosis and has been developed for each of the arboviral encephalitides described here. A fourfold change in specific IgG antibody (2 to 3 weeks after presentation) is also diagnostic. Viral culture of CSF or serum has low yield. CSF PCR tests for specific arboviral diseases are not yet widely available except in specialized research laboratories; serology thus remains the mainstay of diagnosis. As in other arboviral diseases, CSF serology for West Nile virus is the diagnostic test of choice. West Nile virus IgM is detectable in CSF and serum in the majority (greater than 90%) of infected individuals by day 8 of illness. It should be noted that West Nile virus IgM antibody responses may persist for more than 500 days in some individuals; thus serum IgM cannot definitively distinguish between patients who had remote asymptomatic or non-neuroinvasive infection and those with active CNS disease. West Nile virus IgG peaks at 4 weeks postinfection and persists lifelong. CSF PCR is negative in 30% of immunocompetent individuals with neuroinvasive disease, likely reflective of a short-lived viremia which precedes symptom onset. In immunocompromised individuals who may not mount

KEY POINTS:

In areas endemic for California (La Crosse) virus encephalitis (ie, the Midwest and northeast United States, southern Canada), a high proportion of the population appears to have been infected. Eastern equine encephalitis virus produces the most severe illness among arboviruses, with 50% to 70% mortality and neurological sequelae among 80% of survivors. No cases of Western equine encephalitis have been reported in the United States since 1994, with the last epidemic occurring in 1987 in Colorado. The strain of Venezuelan equine encephalitis virus endemic in the southern United States only rarely causes encephalitis.

85

Copyright @ American Academy of Neurology. Unauthorized reproduction of this article is prohibited.

" VIRAL MENINGITIS AND ENCEPHALITIS

KEY POINTS:

Colorado tick fever virus is likely underrecognized as a cause of aseptic meningitis and encephalitis during the summer months among residents of the United States and Canadian Rocky Mountain regions (average 200 reported cases per year). Lymphocytic choriomeningitis virus should be considered in patients with aseptic meningitis who have had exposure to hamsters or rodents, especially if the illness occurs in the fall or winter, and is associated with moderate CSF pleocytosis (up to 1000 WBCs/mm3) and hypoglycorrhachia.

86

an appropriate neutralizing antibody response, CSF PCR may be a useful diagnostic modality. Nucleic acid amplification tests are also highly effective for the purpose of screening blood products and organ tissues donated by asymptomatic infected individuals. In 2004, with the employment of universal West Nile virus screening measures more than 500 infected specimens were intercepted and prevented from entering the donor supply. Since the advent of West Nile virus disease, surveillance programs in the United States have been greatly expanded to detect enzootic arboviral activity in mosquito, sentinel (ie, equine, chickens), or wild bird populations. Widespread, coordinated local and national surveillance and public health measures, including public education, mosquito abatement programs, and screening of blood products have been important control measures to limit disease during periods of epidemic activity. Human risk has been reduced through vector control and modification of human activity patterns (eg, minimization of outdoor activity during evening and nighttime hours, ensuring proper screening of doors and windows, and appropriate use of insect repellents and protective clothing). Irrigation management has also had an impact on the incidence of arboviral diseases. LESS COMMON CAUSES OF VIRAL MENINGITIS AND ENCEPHALITIS Lymphocytic Choriomeningitis Virus LCMV should be considered in patients with aseptic meningitis who have had exposure to hamsters or rodents, especially if the illness occurs in the fall or winter and is associated with moderate CSF pleocytosis (up to 1000 WBCs/ mm3) and hypoglycorrhachia.

Predominant symptoms include fever, headache, and myalgia, described as severe and involving large muscle groups as well as ocular muscles. Some patients have an associated rash (erythematous eruptions on face and trunk, sometimes with desquamation) in conjunction with lymphadenopathy. Following 3 to 5 days of nonspecific illness, fever subsides for 2 to 4 days and then returns with several days of even more severe headache and often frank meningitis. Myopericarditis may occur during this second febrile period. Rare findings during the convalescent phase (1 to 3 weeks into illness) include alopecia, orchitis, or arthritis. Laboratory findings may include leukopenia, thrombocytopenia, abnormal liver function tests, or pulmonary infiltrates. Compared with other viral meningitides, LCMV is more likely to produce a marked (greater than 1000 cells) pleocytosis and hypoglycorrhachia in the CSF. Diagnosis is based on serological evaluation of serum and CSF as well as culture of CSF (and sometimes blood or urine) for LCMV. Mumps Mumps should be considered in unvaccinated children or adolescents with aseptic meningitis, especially if occurring in late winter or early spring, in association with parotitis or orchitis/ oophoritis. Vaccination has greatly decreased the incidence of mumps and mumps-related meningitis in North America, but mumps remains a common cause of meningitis in the late winter or early spring in other parts of the world where the vaccine is not routinely administered. Following vaccine licensing in the United States in December 1967, the incidence of mumps has steadily declinedfrom 212,000 cases in 1964 to a record low of 231 cases reported to the CDC in

Copyright @ American Academy of Neurology. Unauthorized reproduction of this article is prohibited.

2001. Rare cases of mumps vaccineassociated meningitis have been reported but are very uncommon. The presence of orchitis, oophoritis, parotitis, or pancreatitis in an unimmunized patient with aseptic meningitis is suggestive of the diagnosis. Ninety percent of reported cases occur in children less than 14 years of age. Of all patients with mumps parotitis, at most 10% develop clinical meningitis. However, nearly 50% of the cases of mumps meningitis have occurred in the absence of parotitis. A documented history of previous infection or immunization excludes this diagnosis. Hypoglycorrhachia, as well as a left-shifted CSF pleocytosis, may be more common with mumps meningitis than other viral meningitides. Much less frequently, mumps may be complicated by acute encephalitis, which is generally mild, without focal signs, and with low mortality and few sequelae. A still rarer but more severe encephalitic syndrome presenting as a form of immune-mediated postinfectious encephalomyelitis may occur 7 to 10 days after mumps parotitis. A high percentage of these patients exhibit seizures, hemiparesis, and severe obtundation, and the illness has a mortality of 10%. Diagnosis is based on serological analysis of serum and spinal fluid in conjunction with a culture of nasopharyngeal, CSF, urine, and saliva specimens for mumps virus. Other Herpes-Group Viruses Human herpesvirus 6. HHV-6 has recently been identified as another potential cause of aseptic meningitis and focal encephalitis. The frequency of neurological sequelae is unknown at this time. HHV-6 has been associated with cases of meningitis and encephalitis in both children and adults. In children, it is the cause of roseola infantum, or exanthem sub-

itum, and is frequently associated with febrile seizures. Encephalitis can be focal and thus mimic HSV-1 encephalitis. The importance of HHV-6 as a cause of acute encephalitis is still not clear, but recent retrospective studies have indicated approximately 6% of a series of children and adults with encephalitis had HHV-6 genome demonstrable in CSF by PCR analysis. No clinical features characterized those with HHV-6 as different from other viral encephalitides. Diagnostic tests are generally only available in research laboratories and may include PCR of CSF and serum, serum serology, and blood and CSF culture for HHV-6. HHV-6 has also recently been associated with a variant of MS termed Balos concentric sclerosis. Epstein-Barr virus. Focal encephalitis complicates EBV infection in less than 1% of cases of acute infectious mononucleosis. Although focal involvement of brain parenchyma is itself common, no particular characteristic localization is seen. Recovery is usually complete. Other clinical syndromes caused by EBV include meningitis, transverse myelopathy, and Guillain-Barre syndrome. Serum IgM antibody to EBV viral capsid antigen in a patient with meningitis or encephalitis is strongly suggestive of EBV infection. A DNA PCR assay for EBV on CSF is available, but sensitivity and specificity of the assay in patients with CNS disease is unclear. Treatment is symptomatic since acyclovir has only limited activity against EBV. Varicella-zoster virus. Cerebellar ataxia is a common complication associated with primary chickenpox, occurring 1 week after the onset of rash, and is usually benign. It occurs in immunocompetent individuals. Zoster encephalitis, however, tends to occur in immunocompromised patients, may follow skin eruption by days to months or even occur without recognizable

KEY POINTS:

Compared with other viral meningitides, lymphocytic choriomeningitis virus is more likely to produce a marked (greater than 1000 cells) pleocytosis and hypoglycorrhachia in the CSF. Mumps should be considered in unvaccinated children or adolescents with aseptic meningitis, especially if occurring in late winter or early spring, in association with parotitis or orchitis/ oophoritis. Of all patients with mumps parotitis, at most 10% develop clinical meningitis. Hypoglycorrhachia, as well as a left-shifted CSF pleocytosis, may be more common with mumps meningitis than other viral meningitides.

87

Copyright @ American Academy of Neurology. Unauthorized reproduction of this article is prohibited.

" VIRAL MENINGITIS AND ENCEPHALITIS

KEY POINTS:

Human herpesvirus 6 has recently been identified as another potential cause of aseptic meningitis and focal encephalitis. Focal encephalitis complicates Epstein-Barr virus infection in less than 1% of cases of acute infectious mononucleosis. Cerebellar ataxia is a common complication associated with primary chickenpox, occurring 1 week after the onset of rash, and is usually benign. Varicella-zoster virus encephalitis is for the most part a varicellazoster virus vasculopathy affecting small or large vessels of the CNS or both. Cytomegalovirus can cause an acute necrotizing and demyelinating encephalitic process.

88

skin lesion, and is more severe. VZV encephalitis is for the most part a vasculopathy affecting small or large vessels of the CNS or both. Small vessel disease is more common, presenting as headache, fever, vomiting, mental status change, seizure, and focal deficits. An MRI shows ischemic or hemorrhagic infarcts in cortex and subcortical graywhite matter. Large vessel disease (granulomatous arteritis) presents with acute focal deficit developing weeks to months after contralateral trigeminal distribution zoster. Diagnosis can be made by PCR of CSF for VZV DNA and by detection of intrathecal synthesis of VZV antibody, especially of the IgM class. Patients with VZV encephalitis should be treated with IV acyclovir at a suggested dose of 1500 mg/m2/d (equivalent to 30 mg/kg/d in adults, but often up to 60 mg/kg/d in children) divided into 3 doses for 14 days. Cytomegalovirus. Cytomegalovirus can cause an acute necrotizing and demyelinating encephalitic process. It is unusual in an immunocompetent host but should always be considered in patients with acquired immune deficiency syndrome or patients undergoing immunosuppressive therapy. Encephalitis may be accompanied by an acute viral retinopathy, which is diagnostic. In immunocompromised hosts, the distribution of lesions may be diffuse within the cerebral hemispheres. Diagnosis is made by serological analysis of CSF and serum (with IgM class antibody more suggestive of acute infection), as well as PCR of CSF and serum. Recovery of virus from throat, blood, or urine specimens may be supportive but is not diagnostic because of the high rate of asymptomatic intermittent shedding of virus in seropositive patients. Ganciclovir or foscarnet may be used therapeutically. Herpes B virus. This condition is transmitted by monkey bite and results in severe and fatal encephalitis.

Rabies Although cases of rabies encephalitis are very uncommon in North America, rabies should be considered in the differential diagnosis of any person presenting with unexplained rapidly progressive encephalitis. The disease, once instituted, is invariably fatal (with the exception of one case of survival reported in 2004), but appropriate postexposure prophylaxis is highly effective in preventing disease. Approximately 8000 cases of rabies per year are reported in wild and domestic animals in the continental United States and Puerto Rico, with only rare transmission to humans. Only 36 cases of rabies were diagnosed in humans in the United States between 1990 and 2001. Bats and, to a lesser extent, raccoons, foxes, coyotes, and skunks are the primary carriers of rabies in the United States. Rodents and lagomorphs are not carriers. In South and Central America, dogs and cattle are the primary carriers. Epizootic transmission occurs more often as wildlife population density increases; as populations are decimated, transmission declines. Infection in humans occurs following the bite of a rabid animal, with an incubation period of days to months. It is important to recognize that in the United States, many cases of confirmed rabies have a history of bat exposure but no clear history or evidence of being bitten. Rabies has also been transmitted by organ transplantation. In some patients, no exposure history can be elicited. The lack of a known bite or other exposure history does not, therefore, completely exclude the diagnosis of rabies. When infection develops, viral spread occurs by retrograde axonal transport, and the resulting encephalomyelitis is invariably fatal. Although evaluation of brain biopsy tissue by direct immunofluorescent antibody stain against rabies

Copyright @ American Academy of Neurology. Unauthorized reproduction of this article is prohibited.

KEY POINTS:

TABLE 3-10 Virus

Antiviral Agents Available for Viral Central Nervous System Diseases Treatment
Acyclovir 10 mg/kg dose IV every 8 hours 14 to 21 days Higher doses for neonatal encephalitis (20 mg/kg dose IV every 8 hours for 21 days) Oral acyclovir, famciclovir, valacyclovir for meningitis associated with primary genital herpes simplex virus

Herpes simplex viruses 1 and 2

Varicella-zoster virus

Acyclovir 20 mg/kg dose IV every 8 hours ?Famciclovir, valacyclovir

Although cases of rabies encephalitis are very uncommon in North America, rabies should be considered in the differential diagnosis of any person presenting with unexplained rapidly The lack of a known bite or other exposure history does not exclude the diagnosis of rabies. Acyclovir and pleconaril are the two most effective specific antiviral agents available for treatment of viral encephalitis for herpes simplex virus and enteroviral disease, respectively.

Cytomegalovirus

Ganciclovir Foscarnet

Epstein-Barr virus Enterovirus

Acyclovir (limited effectiveness) Pleconaril (compassionate release only) IV immunoglobulin (for hypogammaglobulinemic patients and neonates with sepsis syndrome)

La Crosse virus Measles virus West Nile virus


IV = intravenous.

? Ribavirin Ribavirin Under study: IV immunoglobulin with high titer to West Nile virus, interferon, antisense nucleotides

proteins is the diagnostic gold standard, RT-PCR is also available. Diagnosis can also be attempted by performing direct immunofluorescent antibody staining and RT-PCR on nuchal skin biopsy, corneal smears, serum, or buccal mucosa specimens. No effective treatment is available although survival has been reported in a single patient who was treated with induction of coma in conjunction with ribavirin and amantidine; optimal medical management is prevention by use of postexposure vaccine and immune globulin. ANTIVIRAL THERAPY Acyclovir and pleconaril are the two most effective specific antiviral agents

available for treatment of viral encephalitis for HSV and enteroviral disease, respectively. General supportive care, including close attention to seizure control, antipyretics, monitoring for syndrome of inappropriate secretion of antidiuretic hormone (SIADH), and evidence of increased intracranial pressure, should be instituted in all patients with viral encephalitis. The use of corticosteroids is controversial (Steiner et al, 2005). Specific antiviral therapies are not available at this time for most instances of viral meningitis and encephalitis. However, several effective antiviral agents have become available in the past decade (Table 3-10). The newest of these agents, pleconaril,

89

Copyright @ American Academy of Neurology. Unauthorized reproduction of this article is prohibited.

" VIRAL MENINGITIS AND ENCEPHALITIS

KEY POINTS:

Because of its methods of activation and action, acyclovir is only effective against DNA viruses that encode a thymidine kinase or related enzyme. In the context of herpes simplex virus encephalitis in adults, therapy with intravenous steroids is sometimes employed in an attempt to decrease temporal lobe swelling, which can encroach on the perimesencephalic cistern, resulting in lateral shift and compression of the brain stem.

90

has already been noted above in the discussion of enteroviral disease. The most important of these is acyclovir, which has greatly improved the survival and outcome of patients with herpetic encephalitis. Acyclovir works by inhibition of the viral DNA polymerase, thus interfering with viral replication. The active form of this drug is a triphosphate. The initial phosphorylation step is catalyzed by a virally encoded thymidine kinase, with subsequent phosphorylations by host cell kinases. Because of its methods of activation and action, acyclovir is only effective against DNA viruses that encode a thymidine kinase or related enzyme. For all practical purposes, this limits acyclovirs efficacy to certain herpesviruses including HSV, VZV, and, to a lesser extent, EBV. Acyclovir is not effective therapy for Cytomegalovirus, which is susceptible only to ganciclovir or foscarnet. In adults with HSV encephalitis, therapy with acyclovir has reduced the mortality rate to 19% from 70% in patients treated with placebo and 50% in patients treated with vidarabine. As previously observed, the patients age and level of consciousness at the start of therapy are important prognostic factors. In a small study, a Glasgow Coma Scale score of greater than 6 predicted 100% survival, whereas patients with a Glasgow Coma Scale score of less than 6 had a 100% incidence of severe sequelae or death. In neonatal HSV disease, mortality from disseminated disease has been reduced from 85% to 54% with highdose acyclovir (60 mg/kg/d divided every 8 hours for 21 days); however, 20% of surviving infants have subsequent neurological impairment. The impact of high-dose acyclovir on mortality from isolated CNS disease has been more encouraging, with a reduction from 50% to 14% mortality in treated infants. Unfortunately, two

thirds of treated survivors still have subsequent neurological impairment. Most experts recommend increasing the dose of acyclovir in neonates to 60 mg/kg/d divided every 8 hours for 21 days, based on results of studies that showed improved neurological outcome compared with infants treated with lower doses of acyclovir. The use of CSF PCR for HSV DNA to monitor the efficacy of acyclovir therapy for HSV encephalitis has generated increasing interest. Most patients treated with a standard 14-day IV course of acyclovir will no longer have detectable HSV DNA in their CSF. A recent consensus report on the role of PCR in management of this disease (Lakeman and Whitley, 1995) suggested that patients who still have detectable HSV DNA in CSF at the end of 14 days should receive an additional course of therapy. The availability of PCR techniques that allow accurate quantitation of the amount of HSV DNA present in CSF will undoubtedly facilitate the use of PCR in therapeutic monitoring. The use of IV corticosteroids is controversial in the setting of encephalitis and ADEM. In the context of HSV encephalitis in adults, therapy with IV steroids is sometimes employed in an attempt to decrease temporal lobe swelling, which can encroach on the perimesencephalic cistern, resulting in lateral shift and compression of the brain stem. No clear data are available to definitely support or refute this practice. In the case of ADEM, IV steroids are widely used and have been anecdotally reported to be effective. However, in several studies of sequential patients with or without such therapy, no difference was found in clinical course or recovery. A variety of other immunomodulatory therapies, including immunosuppressive drugs, plasmapheresis, and IV immune globulin, have also been employed, but

Copyright @ American Academy of Neurology. Unauthorized reproduction of this article is prohibited.

definitive controlled clinical trials are lacking. Supportive measures are indicated in all forms of viral encephalitis and include seizure control with standard anticonvulsant regimens, monitoring for signs of increased intracranial pressure, including the use of intracranial pressure bolts in appropriate

cases, and treatment of increased intracranial pressure with standard therapies such as hyperventilation and osmotic diuresis. Monitoring for SIADH is necessary, and the use of hypotonic fluids should be avoided. Should SIADH occur, fluid restriction measures should be instituted. Hyperthermia should be controlled with antipyretics.

REFERENCES

"

Baringer JR. Herpes simplex virus encephalitis. In: Davis LE, Kennedy PGE, eds. Infectious diseases of the nervous system. Oxford: Butterworth-Heinemann, 2000:11391164. An up-to-date and thorough review of the pathophysiology, clinical features, differential diagnosis, management, and prognosis of viral, bacterial, rickettsial, and parasitic encephalitis, including encephalopathy.

"

Calisher CH. Medically important arboviruses of the United States and Canada. Clin Microbiol Rev 1994;7:89116. A review of Western equine encephalitis, Eastern equine encephalitis, St Louis encephalitis, Powassan virus, La Crosse virus, and Colorado tick fever, including geographical distribution, vectors, epidemiology, hosts, transmission, pathogenesis, differential diagnosis, laboratory diagnosis, and treatment. PreWest Nile virus.

"

Corboy JR, Tyler KL. Neurovirology. In: Bradley WG, Daroff RB, Fenichel GM, Marsden CD, eds. Neurology in clinical practice. 3rd edition. Boston: Butterworth-Heinemann, 2000;823840. A thorough review of the clinical syndromes caused by neurotropic viruses, including meningitis, encephalitis, myelitis, polyradiculitis, and myositis, as well as a discussion of the pathogenesis of CNS viral infection, diagnosis, and therapy.

"

Davis LE. Diagnosis and treatment of acute encephalitis. Neurologist 2000;6:145159. Clinically relevant review and discussion of encephalitis, encephalopathy, and acute disseminated encephalomyelitis.

91

"

DeBiasi RL, Tyler KL. Molecular methods for diagnosis of viral encephalitis. Clin Microbiol Rev 2004;17:903925. Extensive review of newer methodologies, including polymerase chain reaction (PCR), for the diagnosis of viral (and other) etiologies of meningoencephalitis. Includes discussion and clinical applications of multiplex and quantitative real-time PCR.

"

DeBiasi RL, Tyler KL. Recurrent aseptic meningitis. In: Davis LE, Kennedy PGE, eds. Infections of the central nervous system. Oxford: Butterworth-Heinemann, 2000:445479. A thorough review of the causes of recurrent aseptic meningitis with a general approach to the patient and evaluative plan, including specific discussion of infectious, idiopathic, tumors/cyst, drug-induced, connective-tissue disorder, and uveomeningitis etiologies.

Copyright @ American Academy of Neurology. Unauthorized reproduction of this article is prohibited.

" VIRAL MENINGITIS AND ENCEPHALITIS

"

DeBiasi RL, Tyler KL. Polymerase chain reaction in the diagnosis and management of central nervous system infections. Arch Neurol 1999;56:12151219. Discussion of the PCR technique with practical considerations and sensitivity and specificity for all available viral CSF PCRs, as well as bacterial, mycobacterial, fungal, rickettsial, and protozoal PCRs, with application to diagnosis of neurological diseases.

"

Glaser CA, Gilliam S, Schnurr D, et al. In search of encephalitis etiologies: diagnostic challenges in the California Encephalitis Project 1998-2000. Clin Infect Dis 2003;36:731742. Report of an extensive study to characterize etiology of over 300 cases of encephalitis.

"

Gutierrez KM, Prober CG. Encephalitis: identifying the specific cause is key to effective management. Postgrad Med 1998;103:123143. Discussion of history and physical examination clues, as well as cerebrospinal fluid (CSF) analysis, magnetic resonance imaging (MRI), and adjunctive studies (culture and PCR) for the differential diagnosis of enterovirus, herpes simplex virus (HSV), and arbovirus encephalitis.

"

Hammer SM, Connolly KJ. Viral aseptic meningitis in the United States: clinical features, viral etiologies, and differential diagnosis. Cur Clin Top Infect Dis 1992;12:125. A broad review of viral aseptic meningitis, including epidemiology and pathogenesis with specific discussion of enteroviral, HSV, mumps, human immunodeficiency virus (HIV), and arbovirus meningitis.

"

Huang C, Morse D, Slater B, et al. Multiple-year experience in the diagnosis of viral central nervous system infections with a panel of polymerase chain reaction assays for detection of 11 viruses. Clin Infect Dis 2004;39: 630635. Summary of findings from 3485 patients who underwent extensive testing for etiology of encephalitis over a 6-year period (New York State).

"

Johnson RT. Part II: acute neurologic diseases. In: Viral infections of the nervous system. 2nd ed. Philadelphia: Lippincott-Raven, 1998:85224. General review.

92

"

Johnson RT. Acute encephalitis. Clin Infect Dis 1996;23:219226. Clinical article reviewing the etiological agents of viral encephalitis (in the preWest Nile era), diseases which masquerade as viral encephalitis, history and physical findings to support diagnosis of specific encephalitides, and discussion of postinfectious encephalomyelitis.

"

Kennedy PG. Viral encephalitis: causes, differential diagnosis, and management. J Neurol Neurosurg Psychiatry 2004;75;i10i15. An excellent general review.

"

Kimberlin DW. Neonatal herpes simplex infection. Clin Microbiol Rev 2004;17:113. The most up-to-date and comprehensive review on all aspects of this topic.

Copyright @ American Academy of Neurology. Unauthorized reproduction of this article is prohibited.

"

Lakeman FD, Whitley RJ. Diagnosis of herpes simplex encephalitis: application of polymerase chain reaction to cerebrospinal fluid from brain-biopsied patients and correlation with disease. National Institute of Allergy and Infectious Diseases Collaborative Antiviral Study Group. J Infect Dis 1995;171:857863. Landmark study describing the data to support HSV PCR as the diagnostic method of choice and gold standard (replacing brain biopsy) for the diagnosis of herpes simplex encephalitis.

"

Lowry PW. Arbovirus encephalitis in the United States and Asia. J Lab Clin Med 1997;129:405411. A thorough review of arboviral encephalitis, including unusual etiologies that occur on other continents.

"

Petersen LR, Marfin AA. West Nile virus: a primer for the clinician. Ann Intern Med 2002;137:173179. Excellent reference for clinicians encompassing all aspects of West Nile virus disease with focus on information since emergence in North America.

"

Ramers C, Billman G, Hartin M, et al. Impact of a diagnostic cerebrospinal fluid enterovirus polymerase chain reaction test on patient management. JAMA 2000;283:26802685. Describes the impact of enteroviral PCR on the diagnosis and clinical management of suspected aseptic meningitis cases, using a retrospective review of 276 pediatric patients for whom diagnostic enteroviral PCR was performed, including impact on clinical outcome, length of hospital stay, and use of antimicrobials and ancillary tests.

"

Roos KL. Pearls and pitfalls in the diagnosis and management of central nervous system infectious diseases. Semin Neurol 1998;18:185196. Detailed description of interpretation of CSF chemistries and cell counts for the diagnosis of viral CNS infection as well as suggested diagnostic evaluation for HSV (including HSV PCR), human herpes virus 6, varicella-zoster virus, enterovirus, and arbovirus CNS infection.

"

Rotbart HA. Viral meningitis and the aseptic meningitis syndrome. In: Scheld WM, Whitley RJ, Durack DT, eds. Infections of the central nervous system. 2nd edition. Philadelphia: Lipincott-Raven, 1997:2339. Thorough discussion of common causes of aseptic meningitis and their current relative incidences (enterovirus, arbovirus, mumps, lymphocytic choriomeningitis virus), with detailed discussion of enteroviral epidemiology, clinical manifestations, typical CSF patterns, and use of PCR for diagnosis.

93

"

Rubeiz H, Roos RP. Viral meningitis and encephalitis. Semin Neurol 1992;12:165177. A thorough general review.

"

Sawyer MH. Enterovirus infections: diagnosis and treatment. Ped Infect Dis J 1999;18:10331040. A review that encompasses the clinical spectrum of nonpolio enteroviral infections, summarizes the experience with PCR diagnosis, gives insight into the impact of rapid diagnosis, and describes new potential therapies including pleconaril.

Copyright @ American Academy of Neurology. Unauthorized reproduction of this article is prohibited.

" VIRAL MENINGITIS AND ENCEPHALITIS

"

Steiner I, Budka H, Chaudhuri A, et al. Viral encephalitis: a review of diagnostic methods and guidelines for management. Eur J Neurol 2005;12:331344. An up-to-date review with consensus-based recommendations encompassing diagnosis, neuroimaging, and medical and surgical management of patients with viral encephalitis.

"

Tyler KL. West Nile virus infection in the United States. Arch Neurol 2004;61:11901195. A thorough and up-to-date review, including epidemiology, diagnosis, clinical features, treatment, and sequelae.

"

Tyler KL. Viral meningitis and encephalitis. In: Braunwald E, Fauci AS, Kasper DL, et al, eds. Harrisons principles of internal medicine. 15th edition. New York: McGraw-Hill, 2001. The most current general textbook review of viral etiologies of meningitis and encephalitis. Tyler, KL. Diagnosis and management of acute viral encephalitis. Semin Neurol 1984;4:480489. A thorough review with tables differentiating epidemiological, historical, clinical, and laboratory features of various etiologies of viral encephalitis.

"

"

Whitley RJ, Gnann JW. Viral encephalitis: familiar infections and emerging pathogens. Lancet 2002;359:507513. Excellent concise review.

"

Whitley RJ, Kimberlin DW. Viral encephalitis. Pediatr Rev 1999;20:192198. General review including discussion of Cytomegalovirus and HSV CNS infection of neonates, as well as arbovirus, HSV, HIV, and rabies encephalitis in older patients.

94

Copyright @ American Academy of Neurology. Unauthorized reproduction of this article is prohibited.

Anda mungkin juga menyukai